LSAT and Law School Admissions Forum

Get expert LSAT preparation and law school admissions advice from PowerScore Test Preparation.

 Administrator
PowerScore Staff
  • PowerScore Staff
  • Posts: 8917
  • Joined: Feb 02, 2011
|
#24387
Complete Question Explanation

Weaken—PR. The correct answer choice is (C)


The stimulus explains that a certain criminal fugitive has a non-infectious skin problem that will need treatment, so police have published a wanted poster in a medical journal. The stimulus then concludes that it would be ethical for a physician who provides care to the fugitive to report him. The stimulus argues that gunshot wounds and infectious diseases must be reported, and those exceptions to confidentiality are ethical.

The stimulus is severely flawed. Presumably, gunshot wounds evidence a deadly potential, and infectious diseases pose a danger to society. The ethics involved in reporting such dangerous conditions to the police are in no way applicable to reporting someone as a consequence of a visit for a non-infectious disease.

Since you are asked to leave the standard exceptions intact, but support the idea that it would be a violation of ethics if a physician responded to the poster, you should focus on the erroneous comparison the stimulus makes between medical conditions that are either cause or consequence of dangers to society and a medical condition that has no inherent relationship to such a danger.

Answer choice (A):This choice would establish that a physician must always yield any information that a law official happens to want, which would strengthen rather than weaken the stimulus, so this choice is wrong.

Answer choice (B): Though this choice may have been attractive, because it effectively destroys the stimulus, it is incorrect. The question stipulates that you must choose the answer that is consistent with the current exceptions (gunshots and infectious diseases), but not the proposed exception. This response goes too far and does not allow the standard exceptions.

Answer choice (C): This is the correct answer choice. This principle does not allow physicians to report identities except in the case of gunshot wounds. That would prevent the physicians from reporting the fugitive while preserving the exception for gunshot wounds. This choice also allow the exception for infectious diseases, because a physician does not have to report a patient’s identity in order to report the occurrence of an infectious disease.

Answer choice (D): Since disallowing physicians from disclosing a patient’s medical condition would prevent physicians from reporting gunshot wounds and infectious diseases, this principle would not allow the required exceptions, so this choice is wrong.

Answer choice (E): If physicians cannot report the medical conditions of their patients to law enforcement, they cannot meet the required exceptions, so this choice is wrong.
 lathlee
  • Posts: 652
  • Joined: Apr 01, 2016
|
#41771
Hi. can you explain using conditional reasoning please since this question and answer choices do contain obvious conditional reasoning natures.
 Adam Tyson
PowerScore Staff
  • PowerScore Staff
  • Posts: 5153
  • Joined: Apr 14, 2011
|
#42407
Most principles are conditional in nature, lathlee, even when the stimulus is not. Much like a Justify the Conclusion question, where you are looking for an answer that makes the premises sufficient for the conclusion, or assumption questions, where the conclusion is sufficient for the correct answer choice, conditional reasoning can be at play in answers even when the stimulus has no conditional elements.

Here, we would not encourage you to pursue a conditional reasoning diagrammatic approach to the question, because the stimulus is not a conditional argument. Instead, prephrase an answer that will allow two things to be consistent with each other: the requirement that physicians report gunshot wounds, and the claim that reporting this particular fugitive to the police would violate medical ethics. This one reads a lot like a Resolve the Paradox question, so perhaps try thinking of it that way. What would allow these two things to coexist?

Yes, the answers are conditional, but diagramming them is not much use to you since you cannot then plug those conditionals back into the argument in some useful way. Rather than forcing yourself to follow a conditional path, use your prephrase to select the answer that tells you what you need in order to reconcile the two claims.

Not everything needs to be tackled conditionally, lathlee, not even when conditional language is present in the answers. Reserve that approach for conditional arguments or fact sets found in the stimulus.
 mrcheese
  • Posts: 32
  • Joined: Jun 27, 2018
|
#58065
I did not choose C because it only mentions gunshot wounds. I didn't equate the idea that mentioned gunshot wounds to police authorities meant they had to reveal the identity of the patient and I did not equate the idea that when reporting a disease that they would definitely not reveal the identity.

C seemed like it was neglecting what was said about "certain infectious diseases." I did not like any of the other choices really, but I definitely did not like C because it seemed incomplete. Is this supposed to be common sense?
 Rachael Wilkenfeld
PowerScore Staff
  • PowerScore Staff
  • Posts: 1358
  • Joined: Dec 15, 2011
|
#60906
Hi Mr. Cheese,

We can assume that the physicians give out the patients identities in the cases of gunshot wounds and certain infectious diseases because the stimulus states that they are exceptions to confidentiality requirements. If they weren't revealing the identity of the patients, they wouldn't be breaking confidentiality requirements anyway. So the fact that those cases are exceptions indicates that the physicians are disclosing the patient's identifying information.

Answer choice (C) didn't need to address certain infectious diseases because it was only dealing with disclosure to police. The infectious diseases were to be disclosed to public health officials. Since the wanted poster was also asking for disclosure to police, the answer choice explains that gun shot wounds are the only reason physicians should disclose patient information to police. This draws the needed distinction between the gun shot wound case and the fugitive case.

Hope that helps!
Rachael
User avatar
 relona
  • Posts: 24
  • Joined: Jul 23, 2021
|
#90331
Administrator wrote: Tue May 10, 2016 6:27 pm Complete Question Explanation

Weaken—PR. The correct answer choice is (C)


The stimulus explains that a certain criminal fugitive has a non-infectious skin problem that will need treatment, so police have published a wanted poster in a medical journal. The stimulus then concludes that it would be ethical for a physician who provides care to the fugitive to report him. The stimulus argues that gunshot wounds and infectious diseases must be reported, and those exceptions to confidentiality are ethical.

The stimulus is severely flawed. Presumably, gunshot wounds evidence a deadly potential, and infectious diseases pose a danger to society. The ethics involved in reporting such dangerous conditions to the police are in no way applicable to reporting someone as a consequence of a visit for a non-infectious disease.

Since you are asked to leave the standard exceptions intact, but support the idea that it would be a violation of ethics if a physician responded to the poster, you should focus on the erroneous comparison the stimulus makes between medical conditions that are either cause or consequence of dangers to society and a medical condition that has no inherent relationship to such a danger.

Answer choice (A):This choice would establish that a physician must always yield any information that a law official happens to want, which would strengthen rather than weaken the stimulus, so this choice is wrong.

Answer choice (B): Though this choice may have been attractive, because it effectively destroys the stimulus, it is incorrect. The question stipulates that you must choose the answer that is consistent with the current exceptions (gunshots and infectious diseases), but not the proposed exception. This response goes too far and does not allow the standard exceptions.

Answer choice (C): This is the correct answer choice. This principle does not allow physicians to report identities except in the case of gunshot wounds. That would prevent the physicians from reporting the fugitive while preserving the exception for gunshot wounds. This choice also allow the exception for infectious diseases, because a physician does not have to report a patient’s identity in order to report the occurrence of an infectious disease.

Answer choice (D): Since disallowing physicians from disclosing a patient’s medical condition would prevent physicians from reporting gunshot wounds and infectious diseases, this principle would not allow the required exceptions, so this choice is wrong.

Answer choice (E): If physicians cannot report the medical conditions of their patients to law enforcement, they cannot meet the required exceptions, so this choice is wrong.
I am confused on the stimulus. It originally says the fugitive has a NONINFECTOUS skin condition, but the exception to physicians breaking confidentiality only talks about gunshot wounds and reporting certain INFECTIOUS skin conditions. When I read the stimulus, it seemed like the physician had no exception to report the fugitive unless we assume that the fugitive had a gunshot wound which I assumed which is why I didn't choose C
 Robert Carroll
PowerScore Staff
  • PowerScore Staff
  • Posts: 1787
  • Joined: Dec 06, 2013
|
#90446
relona,

That's part of the problem with the argument. The author never said that the ethical exceptions already cover this situation. The author instead was trying to argue by analogy - since exceptions are allowed for gunshot wounds and infectious diseases, there's already a precedent for some exceptions. The author wants to extend those established exceptions into a new exception for this situation. The exception for a noninfectious disease does not exist, and the author is trying to argue that it's similar enough to existing exceptions that it should be introduced.

Answer choice (C), instead of expanding exceptions like the author wants, contracts them into gunshot wounds. Since this isn't a gunshot wound case, there is no exception, so the author's argument is weakened.

Robert Carroll

Get the most out of your LSAT Prep Plus subscription.

Analyze and track your performance with our Testing and Analytics Package.